Loophole: Question Stem Examples Flashcards

1
Q

Which one of the following, if true, most seriously weakens the argument above?

A

Weaken

How well did you know this?
1
Not at all
2
3
4
5
Perfectly
2
Q

Which one of the following, if true, most tends to undermine the argument?

A

Weaken

How well did you know this?
1
Not at all
2
3
4
5
Perfectly
3
Q

Which one of the following, if true, most helps to strengthen the argument?

A

Strengthen

How well did you know this?
1
Not at all
2
3
4
5
Perfectly
4
Q

Which one of the following, if true, most strengthens the argument?

A

Strengthen

How well did you know this?
1
Not at all
2
3
4
5
Perfectly
5
Q

Which one of the following principles, if valid, most strongly supports the reasoning above?

A

Strengthen

How well did you know this?
1
Not at all
2
3
4
5
Perfectly
6
Q

Which one of the following principles, if valid, most helps to justify the reasoning in the argument?

A

Strengthen

How well did you know this?
1
Not at all
2
3
4
5
Perfectly
7
Q

The conclusion drawn above follows logically if which one of the following is assumed?

A

Sufficient Assumption

How well did you know this?
1
Not at all
2
3
4
5
Perfectly
8
Q

The argument’s conclusion can be properly drawn if which one of the following is assumed?

A

Sufficient Assumption

How well did you know this?
1
Not at all
2
3
4
5
Perfectly
9
Q

Which one of the following is an assumption that would allow the conclusion above to be properly drawn?

A

Sufficient Assumption

How well did you know this?
1
Not at all
2
3
4
5
Perfectly
10
Q

Which one of the following, if true, is the strongest counter Laila can make to Timothy’s objection?

A

Counter

How well did you know this?
1
Not at all
2
3
4
5
Perfectly
11
Q

Which one of the following, if true, would provide Sina with the strongest counter to Gabriela’s response?

A

Counter

How well did you know this?
1
Not at all
2
3
4
5
Perfectly
12
Q

If the statements above are true, then on the basis of them which one of the following cannot be true?

A

Contradiction

How well did you know this?
1
Not at all
2
3
4
5
Perfectly
13
Q

The information above, if accurate, can best be used as evidence against which one of the following hypotheses?

A

Contradiction

How well did you know this?
1
Not at all
2
3
4
5
Perfectly
14
Q

Which one of the following would it be most helpful to know in order to judge whether what the scientist subsequently learned calls into question the hypothesis?

A

Evaluate

How well did you know this?
1
Not at all
2
3
4
5
Perfectly
15
Q

The answer to which one of the following would be the most helpful in determining whether William’s argument could be logically defended against Marianna’s objection?

A

Evaluate

How well did you know this?
1
Not at all
2
3
4
5
Perfectly
16
Q

The answer to which one of the following questions is most relevant to evaluating the conclusion drawn above?

A

Evaluate

How well did you know this?
1
Not at all
2
3
4
5
Perfectly
17
Q

Which one of the following would it be most relevant to investigate in evaluating the conclusion of the argument?

A

Evaluate

How well did you know this?
1
Not at all
2
3
4
5
Perfectly
18
Q

Which one of the following would it be most relevant to investigate in evaluating the conclusion of the argument?

A

Evaluate

How well did you know this?
1
Not at all
2
3
4
5
Perfectly
19
Q

Which one of the following, if true, does most to explain the surprising result?

A

Resolution

How well did you know this?
1
Not at all
2
3
4
5
Perfectly
20
Q

Which one of the following, if true, most helps to resolve the apparent discrepancy in the information above?

A

Resolution

How well did you know this?
1
Not at all
2
3
4
5
Perfectly
21
Q

Which one of the following, if true, most helps to resolve the apparent paradox?

A

Resolution

How well did you know this?
1
Not at all
2
3
4
5
Perfectly
22
Q

Which one of the following, if true, helps resolve the apparent conflicts described above?

A

Resolution

How well did you know this?
1
Not at all
2
3
4
5
Perfectly
23
Q

Which one of the following most accurately expresses the main conclusion of the argument?

A

Conclusion

How well did you know this?
1
Not at all
2
3
4
5
Perfectly
24
Q

Which one of the following most accurately expresses the overall conclusion drawn in the argument?

A

Conclusion

How well did you know this?
1
Not at all
2
3
4
5
Perfectly
25
Q

If the statement above is true, which one of the following must also be true?

A

Inference

How well did you know this?
1
Not at all
2
3
4
5
Perfectly
26
Q

If the statement above is true, which one of the following must also be true?

A

Inference

How well did you know this?
1
Not at all
2
3
4
5
Perfectly
27
Q

Which one of the following statements follows logically from the statements above?

A

Inference

How well did you know this?
1
Not at all
2
3
4
5
Perfectly
28
Q

Which one of the following can properly be concluded from the information given above?

A

Inference

How well did you know this?
1
Not at all
2
3
4
5
Perfectly
29
Q

If the statements above are true, which one of the following is an inference that can be properly drawn on the basis of them?

A

Inference

30
Q

Which one of the following is most strongly supported by the information above?

A

Most Strongly Supported (MSS)

31
Q

The information above provides the most support for which one of the following statements?

A

Most Strongly Supported (MSS)

32
Q

The statements above, if true, most strongly support which one of the following?

A

Most Strongly Supported (MSS)

33
Q

Which one of the following most logically completes the argument?

A

Fill-In

34
Q

The conclusion of the argument is strongly supported if which one of the following completes the argument?

A

Fill-In

35
Q

Max and Shirin disagree over whether

A

Controversy

36
Q

The statements above provide the most support for holding that Sandra would disagree with Yul about which one of the following statements?

A

Controversy

37
Q

Which one of the following is the point at issue between Aubry and Tai?

A

Controversy

38
Q

The exchange between JT and Carolina most strongly supports the view that they disagree as to

A

Controversy

39
Q

On the basis of their statements, Jim and Pam are committed to agreeing about which one of the following?

A

Agreement

40
Q

If Retta and Han are both sincere in what they say, then it can be properly concluded that they agree that

A

Agreement

41
Q

The argument assumes which one of the following?

A

Necessary Assumption

42
Q

Which one of the following is an assumption required by the argument?

A

Necessary Assumption

43
Q

The argument depends on assuming that

A

Necessary Assumption

44
Q

Which one of the following most accurately describes how the argument proceeds?

A

Method

45
Q

Of the following, which one most accurately describes Tom’s strategy of argumentation?

A

Method

46
Q

The relationship of Denise’s response to Malcolm’s argument is that Denise’s response

A

Method

47
Q

Wendell responds to Domenick’s argument by

A

Method

48
Q

The reference to the complaint of several centuries ago that powerful memory and extemporaneous eloquence were being destroyed plays which one of the following roles in the argument?

A

Argument Part

49
Q

The statement that the law should require explicit safety labels on toys serves which one of the following functions in consumer advocate’s argument?

A

Argument Part

50
Q

The reasoning in the argument is most vulnerable to criticism on the grounds that the argument

A

Classic Flaw

51
Q

Which one of the following most accurately describes a flaw in the reasoning of the argument?

A

Classic Flaw

52
Q

Which one of the following is a questionable technique employed by the producer in responding to the critic?

A

Classic Flaw

53
Q

The scholar’s reasoning is flawed because the scholar presumes without giving sufficient justification that

A

Loophole Flaw

53
Q

The reasoning in the researcher’s argument is questionable in that the argument overlooks the possibility that

A

Loophole Flaw

54
Q

The advertisement’s reasoning is most vulnerable to criticism on the grounds that it fails to consider whether

A

Loophole Flaw

54
Q

Which one of the following conforms most closely to the principle illustrated above?

A

Principle Conform

55
Q

Which one of the following propositions is best illustrated by the situation described in the passage?

A

Principle Conform

56
Q

The reasoning above conforms most closely to which one of the following propositions?

A

Principle Conform

56
Q

Which one of the following propositions is best illustrated by the passage?

A

Principle Conform

56
Q

The reasoning in which one of the following is most similar to the reasoning above?

A

Parallel Reasoning

57
Q

The pattern of reasoning in the argument above is most similar to that in which one of the following?

A

Parallel Reasoning

58
Q

The pattern of reasoning in the argument above is most closely paralleled in which one of the following?

A

Parallel Reasoning

59
Q

Which one of the following arguments exhibits flawed reasoning that is most parallel to that in the argument above?

A

Parallel Flaw

60
Q

The flawed reasoning in which one of the following arguments most closely resembles the flawed reasoning in the argument above?

A

Parallel Flaw

61
Q

Which one of the following is most appropriate as an analogy demonstrating that the reasoning in the argument above is flawed?

A

Parallel Flaw

62
Q

Mistakenly assumes

A

Loophole Flaw Prefix

63
Q

Fails to consider / fails to establish

A

Loophole Flaw Prefix

64
Q

Presumes without providing sufficient justification

A

Loophole Flaw Prefix

65
Q

Takes for granted

A

Loophole Flaw Prefix

66
Q

Overlooks the possibility that

A

Loophole Flaw Prefix